LSAT and Law School Admissions Forum

Get expert LSAT preparation and law school admissions advice from PowerScore Test Preparation.

 Administrator
PowerScore Staff
  • PowerScore Staff
  • Posts: 8916
  • Joined: Feb 02, 2011
|
#47401
Complete Question Explanation
(The complete setup for this game can be found here: lsat/viewtopic.php?t=17028)

The correct answer choice is (D)

This question is similar to question #22, but instead asks for the variables that cannot be first and second, respectively. The first step is to quickly check the R, W, and M Not Laws on the first position. Regrettably, this reveals no violation. Second, check prior hypotheticals to eliminate the answer choices that could be true and are therefore incorrect. The work in question #20 eliminates answer choice (B), and the work in question #21 helps eliminate answer choice (E).

The three remaining answer choices all feature a combination of variables from the group S, T, and U. Given that all three are featured in the first rule, this is surely notable! Let’s examine each:

Answer choice (A): This arrangement conforms to the rule perfectly, and allows U, also a critical variable due to its role in the second rule, to testify relatively early on. Thus, this answer does not appear to create any issues.

Answer choice (C): Placing T and S first and second, respectively, forces U to testify third in accordance with the second rule. By itself this creates no issues, and allow for ample to room to satisfy the second and third rules.

Answer choice (D): This is the correct answer choice. If U and T are first and second, then neither can testify immediately after S. As this violates the first rule, this answer choice cannot be true and must therefore be correct.

Get the most out of your LSAT Prep Plus subscription.

Analyze and track your performance with our Testing and Analytics Package.